I need help ASAP please please please

Answers

Answer 1

Answer:

n=39/5

Step-by-step explanation:

24=5(n-3)

24=5n-15

-5n= -15-24

-5n=39

n= 39/5


Related Questions

Write an explicit formula for the sequence.
-4,7,-10,13,-16

Answers

Step-by-step explanation:

Sequence is

4

,

7

,

10

,

13

,

16

,

.

.

.

a

1

=

4

,

a

2

=

7

,

a

3

=

10

,

.

.

.

If it is Arithmetic sequence,

a

2

a

1

=

a

3

a

2

=

a

4

a

3

& so on

In the given sum,

a

2

a

1

=

7

4

=

3

a

3

a

2

=

10

7

=

3

a

4

a

3

=

13

10

=

3

Since the difference between the successive terms is same and

hence

common difference

d

=

3

Please help me determine the general equation for the graph above as well as solve for a. Thank you.

Answers

Observe that the x coords of the roots of a polynomial are,

[tex]x_{1,2,3,4}=\{-3,0,1,4\}[/tex]

Which can be put into form,

[tex]y=a(x-x_1)(x-x_2)(x-x_3)(x-x_4)[/tex]

with data

[tex]y=a(x-(-3))(x-0)(x-1)(x-4)=ax(x+3)(x-1)(x-4)[/tex]

Now if I take any root point and insert it into the equation I won't be able to solve for y because they will always multiply to zero (ie. when I pick [tex]x=-3[/tex] the right hand side will multiply to zero,

[tex]y=-3a(-3+3)(-3-1)(-3-4)=0[/tex]

and a will be "lost" in the process.

If we observed a non-root point that we could substitute with x and y and result in a non-loss process then you could find a. But since there is no such point (I don't think you can read it of the graph) there is no other viable way to find a.

Hope this helps :)

Yooooo HELPPP
with this question plz

Answers

Answer:

Step-by-step explanation:

(x-2)(x+4)=x^2+4x-2x-8=0=> x =2, x=0

Answer:

A

Step-by-step explanation:

If a $6 per unit tax is introduced in this market, then the new equilibrium quantity will be

Answers

Answer:

soory i dont know just report me if you angry

please help !!!!
i would really appreciate it ​

Answers

Answer: A

Step-by-step explanation: x=-2, y=3, z=-3

Answer:

A. -2, 3, -3

Step-by-step explanation:

x = 7 - 2y + z

y = 21 + 6x + 2z = 21 + 6×(7 - 2y + z) + 2z =

= 21 + 42 - 12y + 6z + 2z = 63 - 12y + 8z

13y = 63 + 8z

y = (63 + 8z)/13

2x + 2y - 3z = 11

2×(7 - 2y + z) + 2×(63 + 8z)/13 - 3z = 11

2×(7 - 2×(63 + 8z)/13 + z) + 2×(63 + 8z)/13 - 3z = 11

14 - 4×(63 + 8z)/13 + 2z + 2×(63 + 8z)/13 - 3z = 11

-2×(63 + 8z)/13 - z = -3

-2×(63 + 8z) - 13z = -39

-126 - 16z - 13z = -39

-29z = 87

z = -3

y = (63 + 8×-3)/13 = (63 - 24)/13 = 39/13 = 3

x = 7 - 2×3 + -3 = 7 - 6 - 3 = -2

I'm interval notation please

Answers

9514 1404 393

Answer:

  (-2, 4]

Step-by-step explanation:

  -21 ≤ -6x +3 < 15 . . . . given

  -24 ≤ -6x < 12 . . . . . . subtract 3

  4 ≥ x > -2 . . . . . . . . . . divide by -6

In interval notation, the solution is (-2, 4].

__

Interval notation uses a square bracket to indicate the "or equal to" case--where the end point is included in the interval. A graph uses a solid dot for the same purpose. When the interval does not include the end point, a round bracket (parenthesis) or an open dot are used.

find the quotient 1/5 / (-5/7) =

Answers

Answer:

-7/25

Step-by-step explanation:

1/5 ÷ (-5/7)

Copy dot flip

1/5 * -7/5

-7/25

If 1100 square centimeters of material is available to make a box with a square base and an open top, find the largest possible volume of the box. Round to two decimal places if necessary.

Answers

volume= a^2 * h

area= a^2+4ah

take the second equation, solve for h

4ah=1100-a^2

h=1100/4a -1/4 a now put that expression in volume equation for h.

YOu now have a volume expression as function of a.

take the derivative, set to zero, solve for a. Then put that value back into the volume equation, solve for Volume.

Cited from jiskha

Ilang litro ng tubig ang kailangang isalin sa timba na naglalaman ng 10 000 mililitro

Answers

Answer

nghiệmTrảingu       từng bước:

If x+y=8 and xy =15 find the value of x³+y³.​

Answers

Answer:

152

Step-by-step explanation:

let x= 5 and y= 3x + y = 85 + 3 = 8xy = 155 × 3 = 15x³ + y³ = ?5³ + 3³ = ?125 + 27 = 152

[tex]\tt{ \green{P} \orange{s} \red{y} \blue{x} \pink{c} \purple{h} \green{i} e}[/tex]

What is the approximate length of arc s on the circle below? Use 3.14 for Pi. Round your answer to the nearest tenth.

-5.8 ft
-6.3 ft
-27.5 ft
-69.1 ft

Answers

9514 1404 393

Answer:

  69.1 ft

Step-by-step explanation:

The diameter of the circle is 24 ft. The length of the arc is more than twice the diameter, so cannot be less than about 50 ft. The only reasonable choice is ...

  69.1 ft

__

The circumference of the circle is ...

  C = 2πr = 2(3.14)(12 ft) = 75.36 ft

The arc length of interest is 330° of the 360° circle, so is 330/360 = 11/12 times the circumference.

  s = (11/12)(75.36 ft) = 69.08 ft ≈ 69.1 ft

Answer:D

Step-by-step explanation:

To calculate the volume of a chemical produced in a day a chemical manufacturing company uses the following formula below:
[tex]V(x)=[C_1(x)+C_2(x)](H(x))[/tex]
where represents the number of units produced. This means two chemicals are added together to make a new chemical and the resulting chemical is multiplied by the expression for the holding container with respect to the number of units produced. The equations for the two chemicals added together with respect to the number of unit produced are given below:
[tex]C_1(x)=\frac{x}{x+1} , C_2(x)=\frac{2}{x-3}[/tex]
The equation for the holding container with respect to the number of unit produced is given below:
[tex]H(x)=\frac{x^3-9x}{x}[/tex]

a. What rational expression do you get when you combine the two chemicals?
b. What is the simplified equation of ?
c. What would the volume be if 50, 100, or 1000 units are produced in a day?
d. The company needs a volume of 3000 How many units would need to be produced in a day?

Answers

Answer:

[tex]V(x) = [\frac{x}{x + 1} + \frac{2}{x-3}] * \frac{x^3 - 9x}{x}[/tex]

[tex]V(x) = [\frac{(x^2-x+2)(x + 3)}{(x + 1)}][/tex]

[tex]V(50) = 2548.17[/tex]        [tex]V(100) = 10098.10[/tex]       [tex]V(1000) = 999201.78[/tex]

[tex]x = 54.78[/tex]

Step-by-step explanation:

Given

[tex]V(x) = [C_1(x) + C_2(x)](H(x))[/tex]

[tex]C_1(x) = \frac{x}{x+1}[/tex]

[tex]C_1(x) = \frac{2}{x-3}[/tex]

[tex]H(x) = \frac{x^3 - 9x}{x}[/tex]

Solving (a): Expression for V(x)

We have:

[tex]V(x) = [C_1(x) + C_2(x)](H(x))[/tex]

Substitute known values

[tex]V(x) = [\frac{x}{x + 1} + \frac{2}{x-3}] * \frac{x^3 - 9x}{x}[/tex]

Solving (b): Simplify V(x)

We have:

[tex]V(x) = [\frac{x}{x + 1} + \frac{2}{x-3}] * \frac{x^3 - 9x}{x}[/tex]

Solve the expression in bracket

[tex]V(x) = [\frac{x*(x-3) + 2*(x+1)}{(x + 1)(x -3)}] * \frac{x^3 - 9x}{x}[/tex]

[tex]V(x) = [\frac{x^2-3x + 2x+2}{(x + 1)(x -3)}] * \frac{x^3 - 9x}{x}[/tex]

[tex]V(x) = [\frac{x^2-x+2}{(x + 1)(x -3)}] * \frac{x^3 - 9x}{x}[/tex]

Factor out x

[tex]V(x) = [\frac{x^2-x+2}{(x + 1)(x -3)}] * \frac{x(x^2 - 9)}{x}[/tex]

[tex]V(x) = [\frac{x^2-x+2}{(x + 1)(x -3)}] * (x^2 - 9)[/tex]

Express as difference of two squares

[tex]V(x) = [\frac{x^2-x+2}{(x + 1)(x -3)}] * (x- 3)(x + 3)[/tex]

Cancel out x - 3

[tex]V(x) = [\frac{x^2-x+2}{(x + 1)}] *(x + 3)[/tex]

[tex]V(x) = [\frac{(x^2-x+2)(x + 3)}{(x + 1)}][/tex]

Solving (c): V(50), V(100), V(1000)

[tex]V(x) = [\frac{(x^2-x+2)(x + 3)}{(x + 1)}][/tex]

Substitute 50 for x

[tex]V(50) = [\frac{(50^2-50+2)(50 + 3)}{(50 + 1)}][/tex]

[tex]V(50) = \frac{(2452)(53)}{(51)}][/tex]

[tex]V(50) = 2548.17[/tex]

Substitute 100 for x

[tex]V(100) = [\frac{(100^2-100+2)(100 + 3)}{(100 + 1)}][/tex]

[tex]V(100) = \frac{9902)(103)}{(101)}[/tex]

[tex]V(100) = 10098.10[/tex]

Substitute 1000 for x

[tex]V(1000) = [\frac{(1000^2-1000+2)(1000 + 3)}{(1000 + 1)}][/tex]

[tex]V(1000) = [\frac{(999002)(10003)}{(10001)}][/tex]

[tex]V(1000) = 999201.78[/tex]

Solving (d): V(x) = 3000, find x

[tex]V(x) = [\frac{(x^2-x+2)(x + 3)}{(x + 1)}][/tex]

[tex]3000 = [\frac{(x^2-x+2)(x + 3)}{(x + 1)}][/tex]

Cross multiply

[tex]3000(x + 1) = (x^2-x+2)(x + 3)[/tex]

Equate to 0

[tex](x^2-x+2)(x + 3)-3000(x + 1)=0[/tex]

Open brackets

[tex]x^3 - x^2 + 2x + 3x^2 - 3x + 6 - 3000x - 3000 = 0[/tex]

Collect like terms

[tex]x^3 + 3x^2- x^2 + 2x - 3x - 3000x + 6 - 3000 = 0[/tex]

[tex]x^3 + x^2 -3001x -2994 = 0[/tex]

Solve using graphs (see attachment)

[tex]x = -54.783[/tex] or

[tex]x = -0.998[/tex] or

[tex]x = 54.78[/tex]

x can't be negative. So:

[tex]x = 54.78[/tex]

Solve 7 ( x + 1 ) + 2 = 5x + 15

Answers

Answer:

x = 3

Step-by-step explanation:

7(x + 1) + 2 = 5x + 15

~Simplify left side

7x + 7 + 2 = 5x + 15

~Combine like terms

7x + 9 = 5x + 15

~Subtract 9 to both sides

7x = 5x + 6

~Subtract 5x to both sides

2x = 6

~Divide 2 to both sides

x = 3

Best of Luck!

Need tha answer explained

Answers

Answer:

Bri what do you mean explanation your answer is correct

Please mark me brainliest thanks

Answer:

It is 77.2, so your anwer is correct.

Step-by-step explanation:

Finding decimal divided by decimal too hard? Don't worry, I've got your back! To do division, you can do it the hard way by just dividing it, but there's something more simple.

Move the dividend's decimal point to the right until it's not a decimal. Do the same with the divisor, but it depends on how many decimal places on the dividend was moved by. So in this case, you move it by 2 decimal places for BOTH! Then you just simply divide it. It gives you the same answer.

BTW if I didn't make my explanation clear, please comment.

help please! I need the answer quickly! thank you!

Answers

Answer:

B) 1 unit to the left

Step-by-step explanation:

Plzzz I’m giving a away 25 points

Answers

Answer:

sin ß = opposite / hypotenuse

sin45° = x / 4√2

Cross multiply

x = sin 45° × 4√2

x = √2/2 × 4√2

x = 4 × √2 ×√2 / 2

x = 4 × 2 / 2

x = 8 / 2

x = 4


[tex] \frac{3x - 2}{7} - \frac{5x - 8}{4} = \frac{1}{14} [/tex]

Answers

Answer:

[tex]x=2[/tex]

Step-by-step explanation:

[tex]\frac{3x-2}{7}-\frac{5x-8}{4}=\frac{1}{14}[/tex]

In order to factor an integer, we need to divide it by the ascending sequence of primes 2, 3, 5.

The number of times that each prime divides the original integer becomes its exponent in the final result.

In here,  Prime number 2 to the power of 2 equals 4.

[tex]\frac{3x-2}{7}-\frac{5x-8}{2^{2} }=\frac{1}{14}[/tex]

First, We need to add fractions-

Rule:-

[tex]\frac{A}{B} +\frac{C}{D} =\frac{\frac{LCD}{B}+\frac{LCD}{D}C }{LCD}[/tex]

LCD = [tex]7 \cdot 2^{2}[/tex]

[tex]\frac{4(3x-2)+7(-(5x-8))}{7*2^{2} } =\frac{1}{14}[/tex]

[tex]x=2[/tex]

OAmalOHopeO

guys pls tell me this answer as soon as possible​

Answers

que es un cuadrilatero

Find the value of x. Round to the nearest tenth.

Answers

Answer:

1.6 ft

Step-by-step explanation:

If you use the Pythagorean Theorem to solve for x, you get:

[tex]x=\sqrt{2.1^2-1.4^2}[/tex]

[tex]x=\sqrt{2.45} = 1.56524758425[/tex]

Rounded to the nearest tenth, the answer is 1.6

Find two consecutive even numbers whose sum is 758.

Answers

Answer:

378 and 380

Step-by-step explanation:

The two even consecutive numbers that add up to 758 are going to be very close to half of 758. This is because two half of 758 are going to be the most similar addends of 758. This is important because the answers will be consecutive and therefore, must also be very similar. To solve, first, divide 758 by 2. This is 379, which is not an even number. So, to find the needed addends subtract and add 1 to 379. Both of these will be even and consecutive. These two numbers are 378 and 380. Then, to check you, can add them and see that they do sum 758.

Answer:

Step-by-step explanation:

Let the first number = x

Let the second number = x + 2

x + x + 2 = 758                      Collect like terms

2x + 2 = 758                         Subtract 2

2x = 758 - 2                          Combine

2x = 756                                Divide by 2

2x/2 = 756/2

x = 378

The first number is 378

The second number 380

If your teacher is really fussy, you can do it this way.

Let the first number = 2x

Let the second number = 2x + 2

The reason for this is to guarantee that both numbers were even to start with.

2x + 2x+2 = 758                    Combine like terms

4x + 2 = 758                          Subtract 2

4x = 756                                Divide by 4

x = 756/4

x = 189

Therefore 2x = 378

2x + 2 = 380                 Just as before.

Simplify the given expression.

Answers

Answer:

8x-21

----------------------

(2x-7)(2x+7)

Step-by-step explanation:

7                       4

-----------   + ------------

4x^2 -49    2x+7

Factor  ( notice that it is the difference of squares)

7                       4

-----------   + ------------

(2x)^2 - 7^2    2x+7

7                       4

-----------       + ------------

(2x-7)(2x+7)    2x+7

Get a common denominator

7                       4(2x-7)

-----------       + ------------

(2x-7)(2x+7)    (2x-7)(2x+7)

Combine

7 +4(2x-7)

----------------------

(2x-7)(2x+7)  

7 +8x-28

----------------------

(2x-7)(2x+7)  

8x-21

----------------------

(2x-7)(2x+7)  

Answer:

(8x - 21) / (2x + 7)(2x - 7)

Step-by-step explanation:

7 / (4x^2 - 49)+ 4 / (2x + 7)

= 7 / (2x + 7)(2x - 7) + 4 / (2x + 7)

LCM = (2x + 7)(2x - 7)   so we have

(7 + 4(2x - 7) / (2x + 7)(2x - 7)

=   (8x - 21) / (2x + 7)(2x - 7).

An advertiser goes to a printer and is charged $36 for 80 copies of one flyer and $46 for 242 copies of another flyer. The printer charges a fixed setup cost plus a charge for every copy of single-page flyers. Find a function that describes the cost of a printing job, if xx is the number of copies made.

Answers

Answer:

ytre

Step-by-step explanation:

A bag contains 3 red, 5 blue, and 4 white marbles. If a marble is drawn from the bag, not replaced, and another is drawn, what is the probability of selecting a white and then blue marble?

Answers

Answer:

5/33

Step-by-step explanation:

The probability of selecting a white marble is 4/12 = 1/3(because we have 4 white marbles and 12 marbles in total )

The probability of selecting a blue marble is 5/11 (because we have 5 blue marbles and 11 marbles left(1 has been drawn already) )

Probability of selecting a white and then blue marble equals 1/3*5/11 = 5/33

WILL GIVE BRAINLIEST
Complete the equation describing how
x and y are related.
х
-3
-2
-1
0
1
2
3
y
12
8
4
0
-4
-8
-12
y = [? ]x

Answers

Answer:

[tex]y=4x[/tex]

Answer:

y = -4x

hope that helped.........

Factor 64a^3 -8b^3 Explain all steps.

Answers

Answer:

[tex]8(2a- b)(4a^2+ 2ab+ b^2)[/tex]

Step-by-step explanation:

factor out the 8

then you have the sum/difference of cubes..

look that up SOAP: same opposite, always a plus

[tex]64a^3 -8b^3\\8(8a^3 -b^3)[/tex]

[tex]8(2a- b)(4a^2+ 2ab+ b^2)[/tex]

what percent of 70 is 35

Answers

Answer:

50%

Step-by-step explanation:

35 is halve of 70 therefore it is 50%

hope it helps u...........

NO LINKS OR ANSWERING QUESTIONS YOU DON'T KNOW!!!

Chapter 11 part 2:

What are three different properties of logarithmic functions when encountering the operations of addition, subtraction, and multiplication? Provide an example of each.

Answers

The three main log rules you'll encounter are

log(A*B) = log(A) + log(B)log(A/B) = log(A) - log(B)log(A^B) = B*log(A)

The first rule allows us to go from a log of some product, to a sum of two logs. In short, we go from product to sum. The second rule allows us to go from a quotient to a difference. Lastly, the third rule allows to go from an exponential to a product.

Here are examples of each rule being used (in the exact order they were given earlier).

log(2*3) = log(2) + log(3)log(5/8) = log(5) - log(8)log(7^4) = 4*log(7)

----------------

Here's a slightly more complicated example where the log rules are used.

log(x^2y/z)

log(x^2y) - log(z)

log(x^2) + log(y) - log(z)

2*log(x) + log(y) - log(z)

Hopefully you can see which rules are being used for any given step. If not, then let me know and I'll go into more detail.

I’m new to this app and I need help with those two questions please help!!

Answers

y=x²-10x-7

a>0 so we will be looking for minimum

x=-b/2a=10/2=5

y=25-50-7=-32

Answer: (5;32)

y=-4x²-8x+1

а<0 so we will be looking for maximum

х=-b/2a=8/-8=-1

у=4+8+1=13

Maximum point (-1;13)

Algebra II Part 1
Choose the expression or equation that correctly represents this information
Rose works eight hours a day for five days a week. How many hours will she work in sa
weeks?
hours = 40 = 6
hours = 40.6
hours = 6 = 40

Answers

Answer:

240 i.e 40*6

Step-by-step explanation:

if rose works 8hrs per day then she works 40 hrs per week (5 days) therefore 40 hrs per 6 weeks =40*6=240

Answer:

40

Step-by-step explanation:

After running 3/4 of a mile tess has only run 1/3 how long is the race in miles but I want to know how you did it

Answers

Take 3/4 and 1/3 multiply the bottom, thats your denominator. Then since you multiplied 4•3, multiply the top by three.
Other Questions
Write an argumentative essay about coronavirus and its impact in PNG and parts of the world. briefly explain the weakness of the RSA's industrial development zones Write the process of the formation of District Co-ordination Committee.Answer it fast plz TutoCombine any like terms in the expression. If there are no like terms, rewrite the expression.8r + 9pg - pg - pq what do you mean by occupation?? in act 2 the lord chamberlian is now no less concerned that laetres comduct in paris does not make him look too good Solve 7 ( x + 1 ) + 2 = 5x + 15 an atom that gained an electron is called _____ stock is the number of shares that a corporation's charter allows it to sell. The number of these shares usually exceeds the number of shares issued (and outstanding), often by a large amount. Which points are possible approximations for this system? Select two options. corto anlisis filosfico sobre qu es alienacin Write balanced equations for the reaction of each of the following carboxylic acids with NaOH. Part A formic acid Express your answer as a chemical equation. A chemical reaction does not occur for this question. Request Answer Part B 3-chloropropanoic acid Express your answer as a chemical equation. nothing A chemical reaction does not occur for this question. o Whats the Difference Between Non-Formal and Informal Learning A sample of gas occupies 1.2 L at 12.0oC. Assuming pressure remains the same, what would be the volume (in L) of this gas at 67oC? A. 1.4 B. 1.0 C. 0.2 D. 5.0 E. 6.7 Need tha answer explained This year, a small business had a total revenue of $42,900 . If this is 35% less than their total revenue the previous year, what was their total revenue the previous year? ac circular runway has a radius of 100 m.Find the distance covered by a person in one complete rotation . The number of bags of Brand Y dog food sold on these five days was 175. Which bar represents the data for Day 5 for Brand Y? Your driver license could be revoked if you are found guilty of: Group of answer choices operating a vehicle without required insurance discharging diesel fumes from the exhaust pipe operating a vehicle with a burned out headlight or tail light discharging a firearm from a vehicle What do is mean by environment friendly behaviour?